Find the limit using Riemann sum

Click For Summary
The discussion focuses on using Riemann sums to find limits, specifically evaluating the limit as n approaches infinity for a series involving fractions. A participant seeks help isolating terms in their limit expression to facilitate the application of Riemann sums. The conversation highlights the potential for Riemann sums with unequal widths and compares two different terms to analyze their limits. Participants share insights on the equivalence of limits for different formulations of the terms involved. The thread concludes with a confirmation of understanding regarding the application of Riemann sums.
devinaxxx

Homework Statement



i want to find limit value using riemann sum
\lim_{n\to\infty}\sum_{i = 1}^{2n} f(a+\frac{(b-a)k}{n})\cdot\frac{(b-a)}{n}= \int_a^b f(x)dx<br>
question : <br>
\lim_{h \to \infty} =\frac{1}{2n+1}+\frac{1}{2n+3}+...+\frac{1}{2n+(2n-1)}<br>

Homework Equations

The Attempt at a Solution


<br>
\lim_{h \to \infty}\sum_{k=1}^n \frac{1}{n}\frac{1}{2+(2k-1)\frac{1}{n}}
i try to isolate 1/n but i can't find way to make this become f(\frac{k}{n}) since k is stuck in 2k-1, can someone give me a hint? thanks
 
Physics news on Phys.org
devinaxxx said:

Homework Statement



i want to find limit value using riemann sum
\lim_{n\to\infty}\sum_{i = 1}^{2n} f(a+\frac{(b-a)k}{n})\cdot\frac{(b-a)}{n}= \int_a^b f(x)dx<br>
question : <br>
\lim_{h \to \infty} =\frac{1}{2n+1}+\frac{1}{2n+3}+...+\frac{1}{2n+(2n-1)}<br>

Homework Equations

The Attempt at a Solution


<br>
\lim_{h \to \infty}\sum_{k=1}^n \frac{1}{n}\frac{1}{2+(2k-1)\frac{1}{n}}
i try to isolate 1/n but i can't find way to make this become f(\frac{k}{n}) since k is stuck in 2k-1, can someone give me a hint? thanks

How different are
$$t_1(n,k) = \frac{1}{2+\frac{2k-1}{n}} \;\; \text{and} \;\; t_2(n,k) = \frac{1}{2 + \frac{2k}{n}} ? $$
Do ##\sum \frac{1}{n} t_1(n,k)## and ##\sum \frac{1}{n} t_2(n,k)## have the same large-##n## limits?
 
  • Like
Likes devinaxxx
Ray Vickson said:
How different are
$$t_1(n,k) = \frac{1}{2+\frac{2k-1}{n}} \;\; \text{and} \;\; t_2(n,k) = \frac{1}{2 + \frac{2k}{n}} ? $$
Do ##\sum \frac{1}{n} t_1(n,k)## and ##\sum \frac{1}{n} t_2(n,k)## have the same large-##n## limits?
thankou got it!
 
Question: A clock's minute hand has length 4 and its hour hand has length 3. What is the distance between the tips at the moment when it is increasing most rapidly?(Putnam Exam Question) Answer: Making assumption that both the hands moves at constant angular velocities, the answer is ## \sqrt{7} .## But don't you think this assumption is somewhat doubtful and wrong?

Similar threads

Replies
17
Views
2K
  • · Replies 3 ·
Replies
3
Views
2K
  • · Replies 1 ·
Replies
1
Views
2K
Replies
7
Views
2K
  • · Replies 8 ·
Replies
8
Views
2K
  • · Replies 7 ·
Replies
7
Views
2K
  • · Replies 3 ·
Replies
3
Views
1K
  • · Replies 1 ·
Replies
1
Views
1K
  • · Replies 14 ·
Replies
14
Views
2K
Replies
6
Views
2K